Sunteți pe pagina 1din 5

MATH 135 W17 Sample Final Exam Solutions

1. What is the greatest common divisor of 1239 and 735?


Answer: 21
Work: We can determine this using the EEA:

1239 = 735 + 504


735 = 504 + 231
504 = 2 · 231 + 42
231 = 5 · 42 + 21
42 = 2 · 21 + 0

2. What is the units digit (also known as the ones digit) of 6789 ?
Answer: 6
Work: We are looking for an integer x such that x ≡ 6789 (mod 10) where 0 ≤ x < 10.
Note that, for all n,
6n ≡ 6 (mod 10)
(We could prove this by induction relying on 62 = 36 ≡ 6 (mod 10).)
Alternatively, using CRT,

6789 ≡ 0 (mod 2)
789
6 ≡1 (mod 5)

Thus 6789 ≡ 6 (mod 10).

3. Give all the solutions to z 3 + 27 = 0 in polar form.


Answer: 3 cos π+2kπ + 3 sin π+2kπ
 
3 3 for k = 0, 1, 2
Work: We have
−27 = 27(cos(π) + i sin(π)).
Thus, CNRT gives us the three roots above.

4. You are eavesdropping on a conversation between Alice and Bob protected by the RSA system. Alice
sends Bob a public encryption key (e, n) and receives a ciphertext C. You are able to determine that
88 < n < 92 and 26 < e < 30. What are the values of n and e?
Answer: (e, n) = (29, 91)
Work: Here n must be the product of two distinct primes. The only possibility is

n = 13 · 7 = 91.

It follows that e must be coprime to (7 − 1)(13 − 1) = 72. The only possibility is

e = 29.
5. Determine all integers x and y such that

• 6x + 10y = 14, and


• x ≡ 3 (mod 7), and
• 20 < y < 40.

Answer: x = −46 and y = 29


Work: The general solutions to the LDE are given by (x, y) = (−1 + 5n, 2 − 3n) where n ∈ Z. Looking
to the second equation, we now have

−1 + 5n ≡ 3 (mod 7).

Thus 5n ≡ 4 (mod 7), which implies


n≡5 (mod 7).
Thus if (x, y) is a solution satisfying the first two equations, there is a k ∈ Z such that

x = −1 + 5(7k + 5) and y = 2 − 3(7k + 5) = −21k − 13.

because 20 < y < 40, it follows that k = −2 is the only possibility.

Alternatively, one could start with x = 7k + 3 and solve the LDE 42k + 10y = −4.

6. Consider the polynomial p(x) = x5 + 2x4 + 6x3 + 12x2 − 27x − 54. One root of this polynomial is 3i.
Express p(x) as a product of five linear polynomials in C[x].
√ √
Answer: p(x) = (x − 3i)(x + 3i)(x + 2)(x − 3)(x + 3)
Work: Since p(x) has real coefficients, CJRT tells us that −3i is also a root of p(x). Therefore

p(x) = (x − 3i)(x + 3i)q(x) = (x2 + 9)q(x)

for some polynomial q(x). Using long division we get that

q(x) = x3 + 2x2 − 3x − 6.

From RRT, any rational root of q(x) will have the form ab , where a | 6 and b | 1. The possibilities are

±1, ±2, ±3, ±6.

Testing these values in q(x) we discover that x = −2 is the only rational root. So

q(x) = (x + 2)q1 (x)

for some polynomial q1 (x). Using long division we get that

q1 (x) = x2 − 3.

We can factor q1 (x) as √ √


q1 (x) = (x − 3)(x + 3).
√ √
Thus p(x) = (x − 3i)(x + 3i)(x + 2)(x − 3)(x + 3).
7. (a) ¬(A ⇒ B) is logically equivalent to (¬A) ⇒ (¬B).
False
(b) ¬(∀x ∈ S, P (x)) is logically equivalent to ∀x ∈ S, (¬P (x)).
False
(c) If a, b, c, d ∈ Z, a | b, b | c, and c | d, then a | d.
True
(d) If a, b, c ∈ Z, and a | bc, then a | b or a | c.
False
(cos π4 + i sin π4 )4 1 i
(e) = +
1−i 2 2
False
(f) A polar form of the complex number z = −18i is 18(cos ( 3π 3π
2 ) + i sin ( 2 )).
True
(g) gcd(299 3100 5101 , 298 3100 5102 ) = 298 3100 5101
True

8. Let a, b, c and d be integers. Prove or disprove each of the following statements.

(a) If ax + by = d has an integer solution, then ax + by + cz = d has an integer solution.


(b) If ax + by + cz = d has an integer solution, then ax + by = d has an integer solution.

Solution:

• This is true. If (x0 , y0 ) is a solution to ax + by = d then (x, y, z) = (x0 , y0 , 0) is a solution to


ax + by + cz = d.
• This is false. Take 2x + 2y + z = 1. This has the solution (x, y, z) = (0, 0, 1) however 2x + 2y = 1
has no integer solution by LDET1 as gcd(2, 2) = 2 which does not divide 1.

9. Prove that if w is an nth root of unity, then w1 is also an nth root of unity.
(Recall: The nth roots of unity are the nth roots of 1.)
Solution: By definition, we have that wn = 1. Conjugating on both sides and using PCJ, we obtain
wn = w n = 1. Thus w n 6= 0. Dividing both sides by w n , we get
 n
1 1
1= n = .
w w

Thus 1/w n is an nth root of unity.

Alternate solution:

If w is an nth root of unity, it follows that |w| = 1 using PM. Thus


1 1·w w
= = = w.
w w·w |w|2

Therefore 1/w is an nth root of unity.


10. Let a ∈ Z. Prove that if gcd(a, 65) = 1, then a12 ≡ 1 (mod 65).
Solution: Since gcd(a, 65) = 1, by GCDPF we see that gcd(a, 5) = 1 and gcd(a, 13) = 1. Then, by SM,
we have that this condition is equivalent to a12 ≡ 1 mod 5 and a12 ≡ 1 mod 13 which we will prove.
Now, a4 ≡ 1 mod 5 by F`T and taking third powers gives a12 ≡ 1 mod 5. Further, a12 ≡ 1 mod 13
by F`T. Combining this and the above gives the result.
az+b
11. Let a, b, c, d ∈ Z such that ad − bc = 1. Let f (z) : C → C be given by f (z) = cz+d . Suppose that z is a
complex number such that Im(z) > 0. Prove that Im(f (z)) > 0.
Solution: Let z ∈ C with Im(z) > 0. Then using PCJ and PM, we have

(az + b)(cz + d)
f (z) =
(cz + d)(cz + d)
(az + b)(cz + d)
=
|cz + d|2
ac|z|2 + bd + adz + bcz
=
|cz + d|2
ac|z|2 + bd + ad Re(z) + bc Re(z) + ad Im(z) + bc Im(z) i
 
= .
|cz + d|2

Since Im(z) > 0, it follows that z 6= 0. From ad − bc = 1, we have that c and d are not both zero. It
follows that cz + d 6= 0 so that |cz + d|2 > 0. Thus

ad Im(z) + bc Im(z) ad Im(z) − bc Im(z) (ad − bc)Im(z) Im(z)


Im(f (z)) = 2
= 2
= 2
= > 0.
|cz + d| |cz + d| |cz + d| |cz + d|2

12. Suppose a, b and n are integers.


Prove that n | gcd(a, n) · gcd(b, n) if and only if n | ab.
Solution:
(=⇒) Suppose
n gcd(a, n) · gcd(b, n).

Since gcd(a, n) a and gcd(b, n) b, we know that

gcd(a, n) · gcd(b, n) ab.

By TD, n ab.

(⇐=) Suppose n ab. Hence there is an integer k so that nk = ab. By EEA, there exist integers x and
y such that
gcd(a, n) = ax + ny,
and integers u and v such that
gcd(b, n) = bu + nv.
Multiplying theses two equations together gives

gcd(a, n) · gcd(b, n) = (ax + ny)(bu + nv)


= abxu + anxv + bnyu + n2 yv
= n(kxu + axv + byu + nyv).

Since (kxu + axv + byu + nyv) ∈ Z, it follows that n gcd(a, n) · gcd(b, n).
n
X
13. Let f (x) = xk be a polynomial in Z2 [x].
k=0
n
X
Prove by mathematical induction that for all n ∈ N, [f (x)]2 = x2k .
k=0
Solution: We prove the statement
" n
#2 n
X X
P (n) : xk = x2k
k=0 k=0

for all n ≥ 1 by induction.

Base case: When n = 1, the LHS is (1 + x)2 which equals 1 + 2x + x2 which is 1 + x2 because we are
working over Z2 . The RHS is also 1 + x2 . So P (1) is true.

Inductive Hypothesis: Assume that P (m) is true for some integer m ≥ 1.


We show that P (m + 1) is true.
Now when n = m + 1,

m+1
!2 m
!2
X X
k
x = ( xk ) + xm+1
k=0 k=0
m
!2 m
!
X X
k k
= x +2 x xm+1 + x2m+2
k=0 k=0
m
!
X
= x2k + x2m+2 by the Inductive Hypothesis and PC
k=0
m+1
X
= x2k
k=0

Therefore P (m + 1) is true and so the result holds for all n ≥ 1 by induction.

14. Let a and b be coprime positive integers.


Prove that {(s, t) ∈ N × N : as + bt = ab} = ∅.
Solution: Suppose for the sake of finding a contradiction that s and t are natural numbers such that
as + bt = ab. Then bt = ab − as, thus bt = a(b − s). By CAD, and because gcd(a, b) = 1, it follows that
b | (b − s). Note that s > 0 and s < b. Hence 0 < b − s < b. This is a contradiction to BBD.
Alternate solution: Consider the LDE ax + by = ab. One solution is given by x = b and y = 0. The
general solutions are given by x = b + bn and y = 0 − an. Note that 0 + an > 0 if and only if n > 0,
and b − bn > 0 if and only if n < 0. Thus there is no solution with both x and y positive.

S-ar putea să vă placă și